Đến nội dung

Hình ảnh

Đề thi chọn đội tuyển HSG QG Hà Nội năm học 2014-2015


  • Please log in to reply
Chủ đề này có 18 trả lời

#1
Cetus

Cetus

    Binh nhất

  • Thành viên
  • 42 Bài viết

Đề thi chọn đột tuyển Hà Nội vòng 2 năm học 2014-2015

Hình gửi kèm

  • 1.jpg


#2
Ispectorgadget

Ispectorgadget

    Nothing

  • Quản lý Toán Phổ thông
  • 2946 Bài viết
ĐỀ THI CHỌN HSG THÀNH PHỐ HÀ NỘI NĂM HỌC 2014-2015
Ngày thi: 28/10/2014
Thời gian làm bài: 180 phút
 
Bài I: (2 điểm)
Xác định tất cả các số tự nhiên $n$ sao cho tồn tại số tự nhiên $m$ để $m^2+9$ chia hết cho $2^n-1$
Bài II: (4 điểm)
Tìm tất cả các hàm số $f:\mathbb{R}^*\to \mathbb{R}^*$ thỏa mãn:
$$f(xf(y)).f(y)=f(x+y)\; \forall x>0,\forall y>0$$
($\mathbb{R}^+$ là tập các số thực dương).
Bài III: (3 điểm)
Với $a,b,c$ là các số thực dương thỏa $a^2+b^2+c^2=2ab+2bc+2ac$, tìm giá trị nhỏ nhất của biểu thức $$P=a+b+c+\frac{1}{abc}-\frac{9}{a+b+c}.$$
Bài IV: (4 điểm)
Cho tứ giác $ABCD$ nội tiếp đường tròn $(O)$ thỏa mãn $AB<BD$ và $CA=CD.$ Gọi $E$ là trung điểm của $AD$ và $I$ là tâm đường tròn nội tiếp tam giác $ABD$. Đường tròn ngoại tiếp tam giác $BIC$ cắt $AB$ tại $F$ ($F$ khác $A,F$ khác $B$). Chứng minh rằng các đường thẳng $AI$ và $EF$ vuông góc với nhau.
Bài V: (4 điểm)
Cho dãy $(u_n)$ xác định bởi $$u_1=2015;u_{n+1}=u_n^2-2014u_n+2014 \; \forall n\in \mathbb{N}$$
Chứng minh với mọi $n$ nguyên dương các số $u_1,u_2,...u_n$ đôi một nguyên tố cùng nhau.
Bài VI (3 điểm)
Cho tập hợp $M=\{-1;0;1\}$. Tìm các bộ số $(a_1;a_2;...;a_{2014})$ thỏa mãn điều kiện $a_i\in M\forall i=\overline{1,2014}$ và $a_i-a_{i+1}\in M \; \forall i=\overline{1,2013}$

 

 

 

--Hết--


►|| The aim of life is self-development. To realize one's nature perfectly - that is what each of us is here for. ™ ♫


#3
luuvanthai

luuvanthai

    Sĩ quan

  • Thành viên
  • 373 Bài viết

 

ĐỀ THI CHỌN HSG THÀNH PHỐ HÀ NỘI NĂM HỌC 2014-2015
Ngày thi: 28/10/2014
Thời gian làm bài: 180 phút

Bài III: (3 điểm)
Với $a,b,c$ là các số thực dương thỏa $a^2+b^2+c^2=2ab+2bc+2ac$, tìm giá trị nhỏ nhất của biểu thức $$P=a+b+c+\frac{1}{abc}-\frac{9}{a+b+c}.$$

Từ gt ta có $a^{2}-2a(b+c)+(b+c)^{2}=4bc\Leftrightarrow (a-b-c)^{2}=(2\sqrt{bc})^{2}\Leftrightarrow a=(\sqrt{b}+\sqrt{c})^{2}$ hoặc $b+c-2\sqrt{bc}=a\Leftrightarrow \sqrt{b}=\sqrt{a}+\sqrt{c}$

Giả sử $\sqrt{a}=\sqrt{b}+\sqrt{c}$

Ta chứng minh $2(a+b+c)\geq 3\sqrt[3]{16abc}$

Thật vậy $2a+2b+2c=a+2(b+c)+(\sqrt{b}+\sqrt{c})^{2}\geq a+8\sqrt{bc}=a+4\sqrt{bc}+4\sqrt{bc}\geq 3\sqrt[3]{16abc}$

Do đó $(a+b+c)^{3}\geq 54abc\$\Rightarrow \frac{1}{abc}\geq \frac{54}{(a+b+c)^{3}}$

Đặt $a+b+c=t\Rightarrow t\geq \sqrt{3}$ vì $(a+b+c)^{2}=4(ab+bc+ac)\leq \frac{4}{3}(a+b+c)^{2}$

Xét hàm $f(t)=t+\frac{54}{t^{3}}-\frac{9}{t};f(t)'=1-\frac{162}{t^{4}}+\frac{9}{t^{2}}=0\Leftrightarrow t=3$

$\Rightarrow f(t)\geq f(3)$

Dấu = xảy ra khi $a=4b=4c;a+b+c=3$


Bài viết đã được chỉnh sửa nội dung bởi luuvanthai: 28-10-2014 - 15:09


#4
Hoang Tung 126

Hoang Tung 126

    Thiếu tá

  • Thành viên
  • 2061 Bài viết

Từ gt ta có $a^{2}-2a(b+c)+(b+c)^{2}=4bc\Leftrightarrow (a-b-c)^{2}=(2\sqrt{bc})^{2}\Leftrightarrow a=(\sqrt{b}+\sqrt{c})^{2}$ hoặc $b+c-2\sqrt{bc}=a\Leftrightarrow \sqrt{b}=\sqrt{a}+\sqrt{c}$

Giả sử $\sqrt{a}=\sqrt{b}+\sqrt{c}$

Ta chứng minh $2(a+b+c)\geq 3\sqrt[3]{16abc}$

Thật vậy $2a+2b+2c=a+2(b+c)+(\sqrt{b}+\sqrt{c})^{2}\geq a+8\sqrt{bc}=a+4\sqrt{bc}+4\sqrt{bc}\geq 3\sqrt[3]{16abc}$

Do đó $(a+b+c)^{3}\geq 54abc\Rightarrow \frac{1}{abc}\leq \frac{1}{54(a+b+c)^{3}}$

Đặt $a+b+c=t\Rightarrow t\geq \sqrt{3}$ vì $(a+b+c)^{2}=4(ab+bc+ac)\leq \frac{4}{3}(a+b+c)^{2}$

Xét hàm $f(t)=t+\frac{54}{t^{3}}-\frac{9}{t};f(t)'=1-\frac{162}{t^{4}}+\frac{9}{t^{2}}=0\Leftrightarrow t=3$

$\Rightarrow f(t)\geq f(3)$

Dấu = xảy ra khi $a=4b=4c;a+b+c=3$

 

 Bài làm của bạn có khá nhiều lỗi sai



#5
mnguyen99

mnguyen99

    Thiếu úy

  • Thành viên
  • 696 Bài viết

 

 
Bài I: (2 điểm)
Xác định tất cả các số tự nhiên $n$ sao cho tồn tại số tự nhiên $m$ để $m^2+9$ chia hết cho $2^n-1$
 

 p là 1 ước nguyên tố của n

Rõ ràng  $2^{p}-1$ tồn 1 ước nguyên tố 4s-1

$\Rightarrow m^{2}+9\equiv 0 (mod 4s-1)$

+Nếu m không chia hết cho 4s-1, áp dụng định lí Fermat ta có điều vô lí.

+$m\vdots 4s-1\Rightarrow 3\vdots 4s-1\Rightarrow 4s-1=3$             

Nên p=2, nếu n có 1ước lẻ ta có điều vô lí

Vậy  n=$2^{k}$


THCS NGUYỄN DUY,PHONG ĐIỀN$\Rightarrow$THPT CHUYÊN QUỐC HỌC HUẾ$\Rightarrow$??? 

 

TẬP LÀM THÁM TỬ TẠI ĐÂY http://diendantoanho...ám/#entry513026


#6
19kvh97

19kvh97

    Sĩ quan

  • Thành viên
  • 423 Bài viết

 

ĐỀ THI CHỌN HSG THÀNH PHỐ HÀ NỘI NĂM HỌC 2014-2015
Ngày thi: 28/10/2014
Thời gian làm bài: 180 phút
 
Bài V: (4 điểm)
Cho dãy $(u_n)$ xác định bởi $$u_1=2015;u_{n+1}=u_n^2-2014u_n+2014 \; \forall n\in \mathbb{N}$$
Chứng minh với mọi $n$ nguyên dương các số $u_1,u_2,...u_n$ đôi một nguyên tố cùng nhau.
 

 

 

 

--Hết--

 

cả đề làm được mỗi câu này : :(

dễ cm $u_n$ là dãy nguyên

Gọi $(u_n;u_{n+1})=d$ từ hệ thức truy hồi suy ra $d \in Ư(2014)$

mặt khác từ hệ thức truy hồi ta có $u_{n+1}-u^2_n \vdots 2014$ do đó $u_n\equiv u^{2^{n-1}}_1\equiv 1 (mod 2014)$

từ 2 điều trên suy ra $d$ chỉ có thể bằng $1$ đpcm



#7
hippotas

hippotas

    Binh nhì

  • Thành viên
  • 11 Bài viết

 p là 1 ước nguyên tố của n

Rõ ràng  $2^{p}-1$ tồn 1 ước nguyên tố 4s-1

$\Rightarrow m^{2}+9\equiv 0 (mod 4s-1)$

+Nếu m không chia hết cho 4s-1, áp dụng định lí Fermat ta có điều vô lí.

+$m\vdots 4s-1\Rightarrow 3\vdots 4s-1\Rightarrow 4s-1=3$             

Nên p=2, nếu n có 1ước lẻ ta có điều vô lí

Vậy  n=$2^{k}$

còn đoạn chứng minh luôn tồn tại m để m2+9 chia hết cho 2n-1 nữa bạn :D 



#8
ChiLanA0K48

ChiLanA0K48

    Trung sĩ

  • Thành viên
  • 133 Bài viết
Bài IV: (4 điểm)
Cho tứ giác $ABCD$ nội tiếp đường tròn $(O)$ thỏa mãn $AB<BD$ và $CA=CD.$ Gọi $E$ là trung điểm của $AD$ và $I$ là tâm đường tròn nội tiếp tam giác $ABD$. Đường tròn ngoại tiếp tam giác $BIC$ cắt $AB$ tại $F$ ($F$ khác $A,F$ khác $B$). Chứng minh rằng các đường thẳng $AI$ và $EF$ vuông góc với nhau.

Gọi $J$ là giao điểm $CE$ và $(O)$

Suy ra $J$ là điểm chính giữa cung $AD$ suy ra $IJ=IA$

Dễ dàng chứng minh $C,O,E,J$ thẳng hàng

$\angle FBI=\angle FCI\Rightarrow \angle FCI= \angle ACE\Rightarrow \left\{\begin{matrix} \Delta IFC\sim \Delta ACE\\\angle AFC=\angle ICE \end{matrix}\right.$

$\Rightarrow \Delta ACF\sim \Delta JCI $

cũng dễ dàng chứng minh $\Delta IFC\sim \Delta JAC$

Từ các tam giác đồng dạng ta có:

$\Rightarrow \left\{\begin{matrix} \frac{AE}{IF}=\frac{CA}{CI}\\ \frac{AF}{IJ}= \frac{AC}{JC}\\ \frac{IF}{JA}=\frac{CI}{CJ} \end{matrix}\right.\Rightarrow AE=AF$

suy ra tam giác $AEF$ cân tại $A$ mà $AI$ là phân giác góc $EAF$ suy ra $AI\perp EF$

 

P/s: Không biết có sai ở đâu ko mong mọi người chỉ giúp



#9
mnguyen99

mnguyen99

    Thiếu úy

  • Thành viên
  • 696 Bài viết

còn đoạn chứng minh luôn tồn tại m để m2+9 chia hết cho 2n-1 nữa bạn :D

Ta sẽ tìm m bằng phương pháp quy nạp:

gọi m là số sao cho $m^{2}\equiv -9 (mod 2^{2^{k}}-1)\Rightarrow m^{2}=(2^{2^{k}}-1)i-9\Rightarrow 2^{2^{k+1}}m^{2}=(2^{2^{k}+2^{k+1}}-2^{2^{k+1}})i-9.2^{2^{k+1}}\Rightarrow (m.2^{2^{k}})^{2}\equiv -9(mod 2^{2^{k+1}}-1)$

áp dụng đồng dư cho phần dư sau.


Bài viết đã được chỉnh sửa nội dung bởi mnguyen99: 29-10-2014 - 19:56

THCS NGUYỄN DUY,PHONG ĐIỀN$\Rightarrow$THPT CHUYÊN QUỐC HỌC HUẾ$\Rightarrow$??? 

 

TẬP LÀM THÁM TỬ TẠI ĐÂY http://diendantoanho...ám/#entry513026


#10
vutuanhien

vutuanhien

    Thiếu úy

  • ĐHV Toán Cao cấp
  • 690 Bài viết

Ta sẽ tìm m bằng phương pháp quy nạp:

gọi m là số sao cho $m^{2}\equiv -9 (mod 2^{2^{k}}-1)\Rightarrow m^{2}=(2^{2^{k}}-1)i-9\Rightarrow 2^{2^{k+1}}m^{2}=(2^{2^{k}+2^{k+1}}-2^{2^{k+1}})i-9\Rightarrow (m.2^{2^{k}})^{2}\equiv -9(mod 2^{2^{k+1}}-1)$

Đoạn cuối suy ra cái đó chưa rõ cho lắm :-?


"The first analogy that came to my mind is of immersing the nut in some softening liquid, and why not simply water? From time to time you rub so the liquid penetrates better, and otherwise you let time pass. The shell becomes more flexible through weeks and months—when the time is ripe, hand pressure is enough, the shell opens like a perfectly ripened avocado!" - Grothendieck


#11
Phuong Thu Quoc

Phuong Thu Quoc

    Trung úy

  • Thành viên
  • 784 Bài viết

cả đề làm được mỗi câu này : :(

dễ cm $u_n$ là dãy nguyên

Gọi $(u_n;u_{n+1})=d$ từ hệ thức truy hồi suy ra $d \in Ư(2014)$

mặt khác từ hệ thức truy hồi ta có $u_{n+1}-u^2_n \vdots 2014$ do đó $u_n\equiv u^{2^{n-1}}_1\equiv 1 (mod 2014)$

từ 2 điều trên suy ra $d$ chỉ có thể bằng $1$ đpcm

Bài này lập luận sai.

$\left ( 2;3 \right )=1;\left ( 3;4 \right )=1;\left ( 2;4 \right )=2$

Có thể làm như sau

Dễ chứng minh $u_{n}\equiv 1\left ( mod2014 \right )\forall n\in \mathbb{N}$

Theo cách xác định dãy thì $u_{n+1}-2014=u_{n}\left ( u_{n}-2014 \right )=u_{n}u_{n-1}\left ( u_{n-1}-2014 \right )=...=u_{n}u_{n-1}...u_{1}\left ( u_{1}-2014 \right )=\prod_{i=1}^{n}u_{i}$

Giả sử tồn tại $i\in \left \{ 1;2;...;n \right \}$ thỏa  $\left ( u_{n+1};u_{i} \right )=d> 1$

Khi đó $u_{n+1}\vdots d;\prod_{i=1}^{n}u_{i}\vdots d\Rightarrow 2014\vdots d\Rightarrow \left ( u_{n+1} ;2014\right )=d> 1$ vô lí

Vậy mỗi số hạng sẽ nguyên tố cùng nhau với tất cả số hạng đứng trước nó trong dãy.

Điều đó có nghĩa tất cả các số hạng của dãy nguyên tố cùng nhau.!

 

Dĩ nhiên $\left ( u_{n} \right )$ là dãy các số tự nhiên.


Bài viết đã được chỉnh sửa nội dung bởi Phuong Thu Quoc: 31-10-2014 - 16:01

Thà một phút huy hoàng rồi chợt tối

 

Còn hơn buồn le lói suốt trăm năm.

 

 


#12
taideptrai

taideptrai

    Trung sĩ

  • Thành viên
  • 103 Bài viết

Ta sẽ tìm m bằng phương pháp quy nạp:

gọi m là số sao cho $m^{2}\equiv -9 (mod 2^{2^{k}}-1)\Rightarrow m^{2}=(2^{2^{k}}-1)i-9\Rightarrow 2^{2^{k+1}}m^{2}=(2^{2^{k}+2^{k+1}}-2^{2^{k+1}})i-9.2^{2^{k+1}}\Rightarrow (m.2^{2^{k}})^{2}\equiv -9(mod 2^{2^{k+1}}-1)$

áp dụng đồng dư cho phần dư sau.

sai rồi bạn ơi! Đoạn cuối bạn suy ra sai rồi. Bạn đọc kĩ lại xem


                                                                                               Nothing is impossible


#13
mnguyen99

mnguyen99

    Thiếu úy

  • Thành viên
  • 696 Bài viết

sai rồi bạn ơi! Đoạn cuối bạn suy ra sai rồi. Bạn đọc kĩ lại xem

Dùng fermat để CM bổ đề sau với x,y không cùng chia hết cho ước số nào có dạng 4k-1 thì $x^{2}+y^{2}$ cũng ko có ước dạng 4k-1.


THCS NGUYỄN DUY,PHONG ĐIỀN$\Rightarrow$THPT CHUYÊN QUỐC HỌC HUẾ$\Rightarrow$??? 

 

TẬP LÀM THÁM TỬ TẠI ĐÂY http://diendantoanho...ám/#entry513026


#14
taideptrai

taideptrai

    Trung sĩ

  • Thành viên
  • 103 Bài viết

Dùng fermat để CM bổ đề sau với x,y không cùng chia hết cho ước số nào có dạng 4k-1 thì $x^{2}+y^{2}$ cũng ko có ước dạng 4k-1.

cái đấy là để cm chiều thuận, còn chiều đảo mới khó, cậu thử cm chiều đảo đi


                                                                                               Nothing is impossible


#15
thukilop

thukilop

    Thượng sĩ

  • Thành viên
  • 291 Bài viết

1962857_663647780419326_8759683150513554


-VƯƠN ĐẾN ƯỚC MƠ-


#16
cachuoi

cachuoi

    Trung sĩ

  • Thành viên
  • 117 Bài viết

tóm lại chưa ai giải đúng bài 1 cả 

bài tổ hợp đếm bằng truy hồi cơ bản 
bài bất đẳng thức có vài cách khác nữa 

bài hàm là bài hay nhất đề



#17
shinichigl

shinichigl

    Trung sĩ

  • Thành viên
  • 135 Bài viết

Đề thi chọn đột tuyển Hà Nội vòng 2 năm học 2014-2015

Bài 2

Nếu tồn tại $y_{0}$ sao cho $f\left (y_{0}  \right )>1$ thì thay $x$ bởi $\frac{y_{0}}{f\left ( y_{0} \right )-1}$ ta có

$f\left ( \frac{y_{0}f\left ( y_{0} \right )}{f\left ( y_{0} \right )-1} \right )f\left ( y_{0} \right )=f\left ( \frac{y_{0}f\left ( y_{0} \right )}{f\left ( y_{0} \right )-1} \right )$. Suy ra $f\left ( y_{0} \right )=1$ (vô lý). Vậy $f\left ( y \right )\leq 1,\forall y>0$

Từ đó ta có $f\left ( y \right )\geq f\left ( x+y \right ),\forall x>0,y>0$. Suy ra $f$ là hàm không tăng

Nếu tồn tại $y_{1}$ sao cho $f\left (y_{1}  \right )=1$ thì thay $y$ bởi $y_{1}$ ta có $f\left ( x \right )=f\left ( x+y_{1} \right ),\forall x>0$

Do $f$ là hàm không tăng và tuần hoàn nên $f$ là hàm hằng. Vậy $f\left ( y \right )=1,\forall y>0$

Ta xét $f\left ( y \right )< 1,\forall y>0$. Suy ra $f$ nghịch biến hay $f$ là hàm đơn ánh.

Thay $x$ bởi $\frac{x}{f\left ( y \right )}$ ta có $f\left ( x \right )f\left ( y \right )=f\left ( \frac{x}{f\left ( y \right )}+y \right ),\forall x>0,y>0$

Hoán đổi vị trí $x$ và $y$ ta được $f\left (\frac{x}{f\left ( y \right )}+y  \right )=f\left ( \frac{y}{f\left ( x \right )}+x \right ),\forall x>0,y>0$

Suy ra $\frac{x}{f\left ( y \right )}+y=\frac{y}{f\left ( x \right )}+x,\forall x>0,y>0$

Như vậy $\frac{1-f\left ( y \right )}{yf\left ( y \right )}=k,\forall y>0$ ($k$ là hằng số, $k\neq 0$)

Suy ra $f\left ( y \right )=\frac{1}{1+ky},\forall y>0$

Thử lại 2 hàm trên, ta thấy đều thõa mãn đề bài.

Vậy $f\left ( y \right )\equiv 1$, $f\left ( y \right )\equiv \frac{1}{1+ky}$


Bài viết đã được chỉnh sửa nội dung bởi shinichigl: 24-02-2015 - 14:13


#18
Uchiha sisui

Uchiha sisui

    Trung sĩ

  • Thành viên
  • 196 Bài viết

Cho em hỏi bài số học giải thế nào ạ ( bài 1)



#19
niemvuitoan

niemvuitoan

    Binh nhất

  • Thành viên mới
  • 35 Bài viết

sai rồi bạn ơi! Đoạn cuối bạn suy ra sai rồi. Bạn đọc kĩ lại xem






3 người đang xem chủ đề

0 thành viên, 3 khách, 0 thành viên ẩn danh